Difference between revisions of "1988 AHSME Problems/Problem 23"

(Added a solution with explanation)
 
Line 10: Line 10:
  
 
==Solution==
 
==Solution==
 
+
By the triangle inequality in <math>\triangle ABC</math>, we find that <math>BC</math> and <math>CA</math> must sum to greater than <math>41</math>, so they must be (in some order) <math>7</math> and <math>36</math>, <math>13</math> and <math>36</math>, <math>18</math> and <math>27</math>, <math>18</math> and <math>36</math>, or <math>27</math> and <math>36</math>. We try <math>7</math> and <math>36</math>, and now by the triangle inequality in <math>\triangle ABD</math>, we must use the remaining numbers <math>13</math>, <math>18</math>, and <math>27</math> to get a sum greater than <math>41</math>, so the only possibility is <math>18</math> and <math>27</math>. This works as we can put <math>BC = 36</math>, <math>AC = 7</math>, <math>AD = 18</math>, <math>BD = 27</math>, <math>CD = 13</math>, so that <math>\triangle ADC</math> and <math>\triangle BDC</math> also satisfy the triangle inequality. Hence we have found a solution that works, and it can be verified that the other possibilities don't work, though as this is a multiple-choice competition, you probably wouldn't do that in order to save time. In any case, the answer is <math>CD = 13</math>, which is <math>\boxed{\text{B}}</math>.
  
  

Latest revision as of 14:20, 27 February 2018

Problem

The six edges of a tetrahedron $ABCD$ measure $7, 13, 18, 27, 36$ and $41$ units. If the length of edge $AB$ is $41$, then the length of edge $CD$ is

$\textbf{(A)}\ 7\qquad \textbf{(B)}\ 13\qquad \textbf{(C)}\ 18\qquad \textbf{(D)}\ 27\qquad \textbf{(E)}\ 36$

Solution

By the triangle inequality in $\triangle ABC$, we find that $BC$ and $CA$ must sum to greater than $41$, so they must be (in some order) $7$ and $36$, $13$ and $36$, $18$ and $27$, $18$ and $36$, or $27$ and $36$. We try $7$ and $36$, and now by the triangle inequality in $\triangle ABD$, we must use the remaining numbers $13$, $18$, and $27$ to get a sum greater than $41$, so the only possibility is $18$ and $27$. This works as we can put $BC = 36$, $AC = 7$, $AD = 18$, $BD = 27$, $CD = 13$, so that $\triangle ADC$ and $\triangle BDC$ also satisfy the triangle inequality. Hence we have found a solution that works, and it can be verified that the other possibilities don't work, though as this is a multiple-choice competition, you probably wouldn't do that in order to save time. In any case, the answer is $CD = 13$, which is $\boxed{\text{B}}$.


See also

1988 AHSME (ProblemsAnswer KeyResources)
Preceded by
Problem 22
Followed by
Problem 24
1 2 3 4 5 6 7 8 9 10 11 12 13 14 15 16 17 18 19 20 21 22 23 24 25 26 27 28 29 30
All AHSME Problems and Solutions

The problems on this page are copyrighted by the Mathematical Association of America's American Mathematics Competitions. AMC logo.png